Soal Kidney Compiled
Soal Kidney Compiled
Soal Kidney Compiled
1
D. Increased anti streptolisin O (ASO),
decreased DNase-B, increased C3 A 55-year-old man refers to ICU after the procedure
E. Increased anti streptolisin O (ASO) open reduction of femur fracture and bleeding intra
(untuk faringitis), increased DNase- operative. At the third day he became apatis, fever 38,5
B(pd nefritik/strepto), decreased C3 C with blood pressure 110/60 mmHg, pulse 55 x/min,
respiratory 20x/min, and urine output decreased to 10
11. A 70-year-old male with tuberculosis, he was cc/hour in 24 hours. At laboratory examination findings
treated with daily streptomycin for 1 week. After the creatinine 4 mg/dL (N: 0.5-1.5), ureum 100 mg/dL (N:
last injection, his urine output diminished and the 20-40), sodium 130 mEq/L (N: 135-155), potassium
laboratory test revealed ureum 80 (20 – 40) mg/dL, serum 7.5 mEq/L (N: 3.5-5.5), blood sugar 200 mg/dL
creatinine 3.0 (0.5 - 1.5) mg/dL. (AIN ada (N: <140).
proteinuria, ATN ga daa proteinuria) From blood gas analysis finding pH 7.1, pCO2 30
What is the most likely diagnosis in this patient? mmHg, pO2 90 mmHg,
HCO3 18 mEq/L. ECG examination finds widening of
A. Urinary tract infection QRS wave.
B. Sepsis
C. Glomerulonephritis 15. What is the most critical condition can cause life
D. Chronic renal failure threatening the patient?
E. Acute tubular nephritis A. nothing for awareness
B. ureum and creatinine at very high lvl
12. What is the most possible cause in this patient? C. hyperkalemia (CKD, AKI)
(AIN hypersentivity, ATN ischemic ata toxin) D. metabolic acidosis
A. Renal ischemia E. severe hyponatremia
B. Direct toxic injury
C. Complex immune response 16. What is the treatment for the critical condition
D. Urinary obstruction above?
E. Drug hypersensitivity A. Waiting for the general condition of the
patient became more better
13. A 3 year old boy was brought to emergency unit with B. Give diuretic until the urine output will
severe diarrhea since three days ago. On arrival he be normally
is not fully alert. His vital sign are as follows: blood C. Hemodialysis immediately (hiperkalemi
pressure is 80/60 mmHg, pulse is not palpable, parah)
respiratory rate was 36 breaths per minute, and D. Give sodium bicarbonate until pH of
body temperature was 38.4oC. On physical exam blood became normal
there is sunken anterior fontanel, dry mucous E. Intubation and ventilator to maintain the
membranes, sunken eyes, lack of tears, poor skin respiratory
turgor, and the capillary refill more then 2 seconds.
The laboratory test in this patient reveal
haemoglobin 13 g/dL(11.5-15.5 g/dL), hematocrit
46% (35-45%), WBC 7,000/mm 3(5,500- 17. A 36-year-old male truck driver is referred to you for
15,500/mm ), 3 platelets count 260,000/mm 3 evaluation of recurrent urolithiasis. The patient has
3
(150,000-400,000 mm ), urea nitrogen 40 mg/dL (5- passed more than 20 calcium oxalate stones since
18 mg/dL), and creatinine 1.7 mg/dL (0.3-0.7 age 18. Previous therapy with hydrochlorothiazide
mg/dL). Urine specific gravity is 1.040 (N: 1.003 – and cellulose phosphate has been ineffective.
1.030) Physical examination is normal. Laboratory finding:
What is the most likely diagnosis in this patient? Serum creatinine 1.0 mg/dL, Serum uric acid 8.9
A. Acute tubular nephritis mg/dL, Urine pH 5.0
B. Prerenal acute kidney injury Radiographs of the abdomen show smooth renal
C. Intrinsic acute kidney injury silhouettes measuring 13.6 cm on the right and 14.2
D. Postrenal acute kidney injury cm on the left, and no radioopaque at kidney region.
E. Chonic renal failure From sonography findings multiple hyperechoic
shadow from both kidney with measure about 2 – 5
14. A 10 year old girl was brought to your clinic with mm.
dysuria since 5 days ago. On physical exam she is Which of the following treatment is the most
healthy with no significant findings. From urine effective for this patient?
culture is found Eschericia coli wih CFU ≥ 100,000. A. Dietary sodium restriction
What is the most likely diagnosis in this patient? B. Dietary calcium supplementation
C. High fluid intake and long-term urine
A. Acute pyelonephritic acidification
B. Reflux nephropathy D. High fluid intake and allopurinol(untuk
C. Uncomplicated urinary tract infection asam urat=radiolucent)
D. Complicated urinary tract infection E. Shock wave lithotripsy
E. Chronic urinary tract infection
2
18. A 75 year old man had urine incontinence since 3 D. Squamous carcinoma
months ago. He had been diagnosed dementia by E. Undifferentited carcinoma
the doctor since 6 months ago. On examination he
had no prostate problems. What is the cause of his 23. A 60-year-old male experiences the onzet of
urination’s problem? headache, naussea ,vomitting fo 1 month. Physical
A. Impaired frontal inhibition examination reveals hypertention, subfebril.
B. Impaired pontine inhibition Pyelogram showed the affected kidney
C. Sympathetic system dysfunction assymmetrically contracted with deformity of the
D. Parasympathetic system dysfunction calyceal system. Gross of the affected kidney
E. Sensoric system dysfunction showed contracted and has irregular granular
surface.The parenchyma is atrophic and replace by
19. A 44 year old man was hospitalized for systemic fibrosis. Microscopic showed glomerular sclerosis,
fungal infection. He was on Amfoterisin B treatment hyalinization and atrophi in cortical area. Another
for 7 days. On examination, he is conscious, BP areas showed fibrosis, chronic inflammatory cells
120/80 mmHg, pulse 80 x/min, RR 16 x/min, with limpocytes agregation.
temperature 36,7 C. Lab: Hb 13 gr%, HT 37%, What is the most likely lesions occur in this kidney?
WBC 8600, Platelet 340.000. Ureum 91, creatinin A. Renal cell carcinoma
2, 0. Volume urine 800 cc/24 jam B. Chronic pyelonephritis
What is the patognomonic laboratory finding in this C. Acute nephritis
case? D. Nephrotic syndrome
A. Increased osmolality E. Diabetic nephropathy
B. Leucocyturia
C. Muddy brown cast 24. A 5-year-old boy child who had a palpable
D. Erythrocyte cast abdominal mass brought to hospital by his mother.
E. Haemoglobinuria The child later had abdominal distention from bowel
obstruction. An USG reveals a 6 cm left renal mass
20. A 24-year-old woman went to a physician because with necrosis and hemorrhage. Microscopic
pain during urination and frequency. On showed embryonal tissue with abortive of tubular
examination, she had has suprapubic pain, but no and glomerular structures.
fever. Her urine was cloudy and shows microscopic What is the diagnosis ot this patient?
evidence of erythrocytes, pyuria, and gram-positive A. Teratoma
bacteria. In the past medical history, she never had B. Retinoblastoma
any symptoms like this one. Which organism is C. Wilms’tumor
most likely responsible for this patient’s illness? D. Hanarthoma
A. Klebsiella pneumonia E. Hemangioma
B. Pseudomonas aeruginosa
25. A 62-year-old woman presents with hematuria and
C. Escherichia coli left flank pain. Physical examination reveals
D. Proteus mirabilis costovertebral angle tenderness. Renal ultrasound
E. Staphylococcus saprophyticus shows marked left hydronephrosis. Serum
creatinine and CBC are normal.
21. A 55-year-old male complains hematuria at the Which of the following diagnostic test is the most
physician’s office for 2 weeks. A cystoscopic relevant?
examinations showed a 2 cm,exophytic,friable A. Renal biopsy
mass on the right bladder dome. B. Renal arteriogram
A biopsy of this lesion show findings most C. Palpation for inguinal lymph nodes
consistent with a/an D. Antinuclear antibody test
A. Papilloma E. IVP
B. Verrucous carcinoma
C. Transitional carcinoma 26. A 34-year-old man comes with 3-day history of low
D. Squamous carcinoma back pain, urinary hesitancy, and pain with
E. Adenocarcinoma urination. He has not had any urethral discharge or
recent sexual contacts. His temperature is 38.2C,Z
22. A 50-year-old male presents with a month long
history of flank pain and hematuria.An USG 27. A 25-year-old man has been crashed by a truck.
abdominal results a 3 cm bosselated mass, The doctor suspects that his kidney is rupture, and
protrudes from cortical are in the right then he sent the patient to have IVP.
kidney.Cytologic examination reveals malignant What is the most likely the appearance of renal
cells. parenchymal rupture in IVP?
What is most likely biopsy finding for this patient? A. The kidney shows non function
A. Wilms tumor B. Ekstravasation (urine bocor )
B. Renal Cell Carcinoma C. Hydronephrosis (urine akumulasi di
C. Transitional carcinoma renal pelvis)
3
D. Normal nephrogram B. Catalase
E. Dilatation of calyx C. Fibronectin
D. Opa protein(go)
28. A 50-year-old man comes to the clinic with complain E. Cord factor (tb)
of abdominal colic. The result of IVP demonstrates
ureteric dilatation. 33. A 67-year-old man with an episode of acute urine
What is the most likely problem? retention was catheterized. Three days later, he
A. Stone in the pelvicureter junction developed fever and suprapubic pain. Culture of
B. Kidney cyst urine revealed a thin film of bacterial growth over
C. Stone in the vesicoureter junction the entire blood agar plate, and urease test was
D. Renal cell Ca stage I positive.
E. Acute inflammation Which of the following is most likely organism to
cause this infection?
29. A 24-year-old woman went to a physician because A. Escherichia coli
of pain during urination. On examination, she had B. Staphylococcus sapropyticus
has left flank tenderness and low-grade fever. Her C. Proteus mirabilis
urine was cloudy and shows microscopic evidence D. Enterococcus faecalis
of erythrocytes, pyuria, and gram-positive bacteria. E. Morganella morgagni
In the past medical history, she never had any
symptoms like this one. 34. A 34-year-old woman presents with pain during
Which organism is most likely responsible for this urination and frequency. On examination, she had
patient’s illness? has suprapubic pain, but no fever. Her urine was
A. Klebsiella pneumonia cloudy and shows microscopic evidence of
B. Pseudomonas aeruginosa erythrocytes, pyuria, and bacteria. The urine has a
C. Escherichia coli high pH. Which of the following bacteria’s strategy
D. Proteus mirabilis is responsible for this condition?
E. Staphylococcus saprophyticus A. Destroy blood vessel in the kidney
B. Exhibits “swarming” motility
30. Which urine specimen is most appropriate for the C. Ferments many sugars
culture examination? D. Produce a potent urease
A. Suprapubic aspiration E. Secretes many exotoxins
B. Mid stream
C. Straight catheter 35. IVP image of a new born baby shows that left kidney
D. Indwelling catheter tremendously enlarged and clearly demonstrates
E. 24 hours urine collection intra-parenchyma mass. What abnormality could
possibly occur?
31. A 30-year-pregnant woman was complaining fever, A. horseshoe kidney
pain when urinating, blood in her urine, and severe B. pelvic kidney
lower back pain. The physician submits her urine for C. unilateral renal agenesis
culture; the next day, the laboratory reports an D. polycystic kidney (ada massa)
organisms growth count of greater than 105/ml of E. pancake kidney
urine. The organism grew well on the blood agar
and producing β–hemolytic colonies. 36. Oligohydramnios can be related to malformation in
Which important virulence factor for the organism is kidney. Why can the amount of amniotic fluid
most likely responsible for this patient’s illness? determine such malformation?
A. Capsule A. baby drinks the amniotic fluid
B. P-fimbriae (Ecoli) B. baby cannot drink amniotic fluid
C. Endotoxin C. amniotic fluid mostly contains urine
D. Phospolipase C D. little amount of amniotic fluid causes
E. Lipooligopolysaccharide the pelvic malformation
E. little amount of amniotic fluid disturbs
32. A 30-year-old man patient complained of a sharp the kidney development
pain in his loin radiating down to his perineum,
accompanied by hematuria. He reported that he 37. A new born baby boy has a malformation in
had a few days of urinary urgency, frequency, and genitourinary appearance. When he urinates, the
burning when urinating. On examination, his urine comes out from large area below the urethra
temperature was 37.5ºC; other vital sign were opening. What process could possible failed during
normal; and he appears to be in pain. A urinalysis the development of genitourinary process?
shows erythrocytes, leukocytes esterase and a high A. closure of genital fold
pH. B. closure of genital opening
Which antigenic composition is most likely C. closure of genital tubercle
responsible for this illness? D. closure of urogenital sinus
A. Hemolysin (proteus) E. closure of Wollfian duct
4
being examined, during an episode his blood
38. Cryptorchid condition results in germ cells failed to pressure is 210/110 mm Hg. Physical examination
multiply, and then die and only in Sertoli cells exist shows no other abnormalities. Urine studies show
in seminferous tubules. What is the development increased catecholamine concentrations. A CT
process the main cause of cryptorchid? scan of the abdomen is most likely to show a mass
A. inguinal hernia in which of the following locations?
B. failure of testicular decent A. Adrenal glands (aldosterone dan
C. failure of gubernaculums regression catecolamin dan adrenalin)
D. fusion of the tunica albuginea and B. Ureter
peritoneum C. Urethral
E. guevedoces D. Bladder
E. Right kidney
39. A 30-year-old man had nausea, vomiting, and
severe colicky right flank pain radiating into the 43. A 6-year-old boy has a large intra-abdominal mass
thigh for 4 hours. He is afebrile. There is right in the midline just above the symphysis pubis.
costovertebral angle tenderness. Urinalysis shows During an operation, a cystic mass is found
numerous RBCs and no bacteria. attached to the umbilicus and the apex of the
Which of the following is the most likely diagnosis? bladder. Which of the following is the most likely
A. Acute glomerulonephritis diagnosis?
B. Bacterial cystitis A. Hydrocele
C. Benign prostatic hyperplasia B. Meckel cyst
D. Urolithiasis C. Meckel diverticulum
E. Renal cell carcinoma D. Omphalocele
E. Urachal cyst
40. A previously healthy 72-year-old man comes to the
physician because of decreased urinary output 44. A 69-year-old man with 82 kg is referred to you with
during the past 2 days; he has had no urinary output fatigue and laboratory result shows Hb 7 g/dL,
for 8 hours. Examination shows suprapubic fullness ureum 55 mg/dL, and creatinin 5.4 mg/dL. He has
and enlarged prostate. not seen a physician for several years.
Which of the following is the most appropriate next Which finding would help you decide that this
step in management? condition is chronic?
A. PSA examination A. Low serum calcium
B. Ultrasonography of the kidneys B. Low serum bicarbonate
C. CT scan of the abdomen C. Protein detected in urinalysis
D. Bladder catheterization D. Broad waxy casts in urinalysis
E. Intravenous pyelography E. Elevated serum phosphorus
41. An otherwise healthy 20-year-old woman is given 45. A 52-year-old woman with longstanding type II
sulfamethoxazole to treat a bladder infection. Three diabetes is referred with microalbuminuria and
days after beginning the antibiotic regimen, the normal serum creatinine. Her weight is 85 kg and
patient has moderately severe jaundice and dark height is 160 cm. Blood pressure is 150/100 mmHg.
urine. Pain with urination and a low-grade fever Which of the following measures should you advise
have resolved. Her hematocrit is 20%. Substantial to prevent progression to overt nephropathy?
numbers of erythrocytes contain Heinz bodies. Her A. Tight glucose control
condition worsens until day 6 of antibiotic therapy, B. High-protein and low carbohydrate diet
when it begins to resolve. Symptoms are C. Blood pressure control
completely gone by day 9 of continued antibiotic D. Regular diuretic
therapy. E. Increased water intake
Which of the following conditions is the most likely
explanation for these findings? 46. A 20-year-old man presents with malaise, nausea,
A. Aplastic anemia and decreased urine output. He was previously well
B. Generalized cytochrome-b5 reductase and his physical examination is normal except for
deficiency an elevated jugular venous pressure (JVP) and
C. Glucose-6-phosphate dehydrogenase pericardia rub. His electrolytes reveal acute renal
deficiency failure (ARF).
D. Pyruvate kinase deficiency Which of the following findings on the urinalysis is
E. Steven Johnson Syndrome most likely in keeping with acute
glomerulonephritis?
A. Granular casts (dan epithelial = ATN)
42. A 61-year-old man comes to the physician because B. Erythrocyte casts
of a 3-month history of episodes of headache, C. Proteinuria
palpitation, and excessive sweating. He has had a D. Hyaline casts (prerenal)
10-kg (22-lb) weight loss during this period. While E. White blood cell casts
5
51. According to the case above, which of the following
47. A 35-year-old women presents with nausea and is the most likely pathophysiology that caused of
vomiting of 2 days duration. She is not on any swelling?
medications and was previously well until now. Her A. Infection
physical examination is normal except for postural B. Hypercholesterolemia
drop in her blood pressure from 110/80 mmHg C. Hypoalbuminemia
supine to 90/80 mmHg standing. Her serum D. Proteinuria
electrolytes are Sodium 130 mEq/L, Potassium 3 E. Water retention
mEq/L, Chloride 90 mEq/L, bicarbonate 30 mEq/L,
urea 50 mg/dL, and creatinine 0.8 mg/dL. 52. A 39-year-old woman with a history of recurrent
Which of the following electrolytes in the case urinary tract infections and now suffering that
above is most likely to be filtered through the infection. Culture examination of urine samples
glomerulus but unaffected by tubular secretion? showed Escherichia colli as the cause of infection.
A. Potassium Patients were given doses of ciprofloxacin in the
B. Urea treatment and symptoms of infection disappear.
C. Creatinine Which of the following is the most correct process
D. Bicarbonate or enzyme that inhibited by the drug?
E. Sodium A. Cell membrane synthesis
B. Cell protein synthesis
48. A 68-year-old man came seeing a general C. Dehidrofolate sintetase
practitioner with a swelling throughout the body. He D. DNA gyrase
also felt urine volume decreases and the urine color E. DNA polimerase
was red. He denied any pain on urination or in any
part of his body, on physical examination 53. A 74 years old man was admitted to the hospital for
costovertebrae angular test negative. Blood test swollen ankle, hands and eyelids. Her skin was
results obtained ureum 55 mg/dL (normal 20-40 pale, damp and cool. The patient’s vital signs were:
mg/dL) and creatinine 1.7 mg/dL (normal <1.1 blood pressure 170/90 mmHg, heart rate 100
mg/dL). Obtained urine lab protein 2 + and 50-60 beats/min, deep respiration 18 breath/min, and
red blood cells. temperature 37.8°C. The patient had no urine
Which of the following diuretics is the most output and the laboratory report showed that the
appropriate for treating the patient above? patient’s blood urea nitrogen [BUN], creatinine,
A. Acetazolamide potassium and phosphate level s was all higher
B. Furosemide than normal. A chest x-ray showed white, fluffy
C. Hidrochlorothiazid patches that spread outward from the hilar areas to
D. Spironolactone the peripheral border of both lungs. Her left ventricle
E. Triamteren appeared moderately enlarged. Which of the
following is the most likely caused of an altered
renal function in this case?
49. A 10 month pregnant woman complained lower A. Elevated BUN level
abdominal pain, and suffering from dysuria for 48 B. No urine output
hours. Patient has mild fever, urine analysis C. Heart failure
showed 1+ protein, but did not find any glucose, and D. Pulmonary disease
blood. Gram stain shows a gram negative bacteria. E. Infection
Which of the following drugs is the most appropriate
for this patient condition? 54. A 50-year-old woman went to a Puskesmas
A. Chloramphenicol complaining of edema in both legs which became
B. Cefixime (ibu hamil), ga bleh quinolone worse along the day. Her Blood pressure is 160/95
C. Cephadroxil mmHg, pulse rate 70/min. She also had a history of
D. Cotrimoxazole an ischemic stroke. Laboratory test are,fasting
E. Tetracycline blood sugar 150 mg/dL (N: <121 mg/dL), serum
creatine 2.1mg/dL (N: <1.1 mg/dL), ureum
50. A 6-year-old girl, brought by her mother to the clinic 100mg%, urinary protein ++. (CKD)
with complaints of swelling that begins from both of Which of the following is the most appropriate drug
eyelid throughout her body. Blood tests obtained: should be given to the patient in order to prevent
albumin 2 g / dL, total cholesterol 250 mg / dL. On secondary stroke attack?
examination of urine obtained protein +3. Which of A. Propranolol
the following is the most likely diagnosis for the case B. Captopril
above? C. Reserpin
A. Nephritic syndrome D. Hydrochlorothiazide
B. Nephrotic syndrome E. Furosemid
C. Acute renal failure
D. Chronic renal failure 55. A 63-year-old man, came to a clinic with of pain on
E. Glomerulonefritis her knees, there are also looked swelling and
6
redness at her both of knees. She said that she also Which of the following lesions could likely to happen
suffering edema in both legs which became worse in this kidney?
along the day. Her Blood pressure is 180/100 A. Renal cell carcinoma
mmHg, pulse rate 90/min. Laboratory test result: B. Chronic pyelonephritis
Hb: 10.8 gram%, serum creatine 2.7mg/dL (N: <1.1 C. Acute nephritis
mg/dL), ureum 80mg%, urinary protein ++. You are D. Nephrotic syndrome
suggesting giving an aspirin (NSAID) h3X500 mg E. Diabetic nephropathy
with caution.
Which of the following is the most reason that you
give the drug with caution? 60. A 5-year-old boy child who had a palpable
A. It is ineffective abdominal mass was brought to hospital by his
B. It causes hypokalemia mother. The child later had abdominal distention
C. It worsen anemia from bowel obstruction. An USG revealed a 6cm
D. its metabolite is nephrotoxic left renal mass with necrosis and hemorrhage. The
E. It decrease kidney filtration microscopic showed embryonal tissue with abortive
of tubular and glomerular structures.
56. A 55-year-old male complained at the physician’s The depicted lesion will most likely be,
office of his hematuria for 2 weeks now. Urine A. Teratoma
cytologic examination showed malignancy cells. A B. Retinoblastoma
cystoscopic examination found a 2 cm, exophytic, C. Wilms’tumor
friable mass on the right bladder dome. D. Hanarthoma
A biopsy of this lesion will likely to show findings that E. Hemangioma
mostly consistent with,
A. Papilloma 61. A 45 years old man, came to the hospital. And told
B. Verrucous carcinoma the doctor that he had episodic severe pain in the
C. Transitional carcinoma left abdomen and radiated into his flank and
D. Squamous carcinoma sometimes into his testical (ipsilateral). From USG
E. Adenocarcinoma the doctor found 6 mm stone in his urinary tract.
57. Which of the following risk factors is most likely to What location is the most possible for the
have led to development of this lesion? obstruction?
A. Diabetes mellitus A. Upper ureter
B. Recurrent urinary tract infection B. Middle ureter
C. Therapy with methicillin C. Lower ureter
D. Cigarette smoking D. Vesicae urinariae
E. Tuberous sclerosis E. Urethra
58. A 50-year old male was present with a month-long 62. If the patient felt the pain radiated from left abdomen
history of flank pain and hematuria. An USG to the testicle cause of a stone in his ureter, what
abdominal imaged a 3-cm bosselated mass which level of sympathetic fibers is the most responsible
protruded from cortical in the right kidney. The for ureter?
cytologic examination revealed malignant cells. A. T 7-8
A biopsy of this mass will likely to show findings that B. S 2-4 (parasimpatik)
mostly consistent with, C. T 11-12 (simpatik bladder)
A. Wilms tumor D. L1-3
B. Renal Cell Carcinoma E. L4-5
C. Transitional carcinoma
D. Squamous carcinoma 63. A 50-year-old man came to his doctor and told the
E. Undifferentited carcinoma doctor that for the last 2 months, he felt burning on
urination and the color was dark. The doctor told
59. On physical examination to a 60-year old male him to check his urine. And the result:
revealed hypertension and subfebril. He has had pH :7 (N : 4.5 – 8) , WBC: 15-20/HPF (N : 0-2/HPF)
history of nausea, vomit and right lateral abdominal ; RBC : 5-10/HPF (N: 0/HPF ). Urine sediments
pain for 2 months. Pyelogram showed that the right show crystals with cuboid shape. (ca oksalat)
kidney smaller than of the left, asymmetrically What is the most likely metabolic abnormality?
contracted with deformity of the calyceal system. A. Hyperoxaluria
The right nephrectomy was performed which B. Hypercalciuria
showed contraction and it has irregular granular C. Hypercitraturia
surface. The parenchyma is atrophic and replaced D. Hyperuricuria
by fibrosis. The microscopic showed glomerular E. Renal tubular acidosis
sclerosis, hyalinization and atrophy in cortical area.
Another area showed fibrosis, with lymphocytes 64. A 42-year-old man came to the hospital and said
aggregation. that his urine’s color is red and he saw vermiform
7
clot. From his statement, could you tell the possible the most likely underlying mechanism in this
source of his problem? patient?
A. Ureter What is the principal management in this patient?
B. Urethra A. Albumin 0,5-1 g/kg, iv, follow with
C. Bladder furosemide 1-2 mg/kgBB/iv
D. Prostate B. Prednisone 40 mg/m2/day
C. Prednisone 60 mg/m2/day
65. A 50-year-old man is hospitalized for acute D. Reduce sodium intake
myocardial infarction. He has decreased cardiac E. Alkylating agent
output with hypotension requiring multiple pressor
agents. His urine output drops over the next 3 days. 69. A 5 day old baby was hospitalized due to indirect
His serum urea nitrogen increases to 59 mg/dL, with hyperbilirubinemia. On the third day hospitalization
creatinine of 2.9 mg/dL. Urinalysis reveals no he had fever and then he was given amoxicillin
protein or glucose, a trace blood, and numerous (beta lactam yang bikin AIN) and gentamycin
hyaline casts. Five days later, he develops polyuria intravenously. Three days later his urine output
and his serum urea nitrogen declines. diminished and the laboratory test revealed urea
Which of the following pathologic findings in his nitrogen 25 (3-12) mg/dL, creatinine 0.9 (0.2-0.4)
kidneys is most likely to have caused his azotemia? mg/dL.
A. Patchy tubular necrosis What is the most likely diagnosis in this patient?
B. Fusion of podocyte foot processes A. Urinary tract infection
C. Glomerular crescents B. Sepsis
D. Hyperplastic arteriolosclerosis C. Acute renal failure
E. Mesangial immune complex deposition D. Chronic renal failure
E. Acute tubular nephritis
66. A 26-year-old diabetic woman is seen in the ER for
sore throat. Rapid strep test is positive for A 3 year old boy was brought to emergency unit
streptococcal pharyngitis and she was started on Karawaci Hospital with severe diarrhea since three days
ampicillin 500 mg four times a day. Three days later, ago. On arrival he is not fully alert. His vital sign are as
she develops hematuria associated with a low follows: blood pressure is 80/60 mmHg, pulse is not
grade fever. On physical examination, she has a palpable, respiratory rate was 36 breaths per minute,
maculopapular rash and a temperature of 38 C. and body temperature was 38.4oC. On physical exam
Laboratory studies show: serum creatinine 3.6 there is sunken anterior fontanel, dry mucous
mg/dl, WBC 8,700 with 56% PMN, 25% lymphs, 3% membranes, sunken eyes, lack of tears, poor skin
monos and 15% eosinophils. Urinalysis: pH 6.2, turgor, and the capillary refill more then 2 seconds. The
protein 2+, blood 3+, 65 RBCs/HPF, 20-30 daily urine volume is less than 400 mL.
WBCs/HPF, 3-4 WBC casts/HPF. ASTO value is 70. In this patient, the tubular epithelial cells are
increased. particular susceptible to ischaemic injury due to:
What is the most likely diagnosis? A. Little charged surface for reabsorption
A. Diabetic nephropathy B. Low metabolic rate and oxygen
B. IgA nephropathy consumption
C. Acute interstitial nephritis C. Decreased levels of salt reaching the
D. Acute pyelonephritis distal tubules
D. Loss of polarity that cause
67. A 40-year old man with arthritis has been using redistribution of membrane proteins
ibuprofen (nonsteroidal anti-inflammatory) every 6 E. Passive transport systems for ions and
hours. He developed renal failure associated with organic acids
nephrotic range proteinuria.
What is the most likely glomerular lesion in this 71. The laboratory test in this patient reveal
patient? haemoglobin 15 g/dL( 11.5-15.5 g/dL), hematocrit
A. Membranoproliferative 46% (35-45%), WBC 7,000/mm 3 (5,500-
glomerulonephritis 15,500/mm3), platelets count 260,000/mm 3
B. Minimal change disease (150,000-400,000 mm3), urea nitrogen 40 mg/dL (5-
C. IgA nephropathy 18 mg/dL), and creatinine 1.7 mg/dL (0.3-0.7
D. Diabetic nephropathy mg/dL).
E. Focal segmental glomerulosclerosis The most likely diagnosis in this patient is:
A. Acute tubular nephritis
68. A 4-year-old-boy was brought to hospital with B. Prerenal acute kidney injury
puffiness around the eyes, especially in the morning C. Intrinsic acute kidney injury
and swelling over the legs. His mother also D. Postrenal acute kidney injury
complaint he has foamy urine. Urine sample shows E. Chonic renal failure
protein ++++ (>3.5 per 1.73 m 2 per 24 hour),
albumin level 1.5g/dL (normal=3.5-5g/dL). What is A 10 year old girl was brought to your clinic with disuria
since 5 days ago. On physical exam she is healthy with
8
no significant findings. From urine culture is found presumed due to a tumor. What of the following is
Eschericia coli with CFU ≥ 100,000. characteristic of this condition?
72. What is the most likely diagnosis in this patient? A. Elevated serum sodium levels
A. Acute pyelonephritic B. Elevated serum potassium levels
B. Reflux nephropathy C. Elevated urinary cortisols
C. Uncomplicated urinary tract infection D. Elevated urinary bicarbonate
D. Complicated urinary tract infection
E. Chronic urinary tract infection 78. An 18-year-old male presents to emergency
following a motorcycle accident. There is significant
73. Factors unfavorable to bacterial growth in this blood loss. In the ER, his BP is 90/60 mmHg and
patient’s urine is: his pulse is 120 x/min. What physiological response
A. A low pH occurs in the kidney?
B. A low concentration of urea A. Decreased urine osmolality
C. The absence of organic acids from a B. Reduced release and action of ADH
diet C. Beta adrenergic stimulation of the
D. A low osmolality vasculature
E. The presence of normal microorganism D. Vasoconstriction of the renal arterioles
E. Reduced aldosterone production and
74. An 8 years old boy presents with decreased urine sodium loss
output, hematuria, high blood pressure, proteinuria
add rising creatinine. One week ago he had a fever 1. Mr. Edi, 42 year old male, comes to the Siloam
and sore throat. A renal biopsy is carried out hospital with muscular weakness, mild headache
reveals diffuse proliferative glomerulonephritis. after eating salted fish. Previously he never eats
Electron microscope will show abnormal deposits in salted fish. Usually he drink only small amount of
what location? water. He said that usually his blood pressure is
A. Between basement membrane and 100/70 mmHg. His blood pressure 140/90 mmHg.
endothelial cells of the glomeruli The doctor sends him to the laboratory. The result
B. Between the basement membrane and shows serum sodium is 150 mEq/L, potassium is
epithelial cells of the proximal tubule 3.5 mEq/L, glucose 150 mg/dl.
C. Between the basement membrane and In this condition, what will the kidney do?
epithelial cells of the glomeruli A. Increase GFR
D. Within the juxtaglomerular apparatus B. Increase obligatory water reabsorption
C. Increase facultative water reabsortion
75. A 45 years old female has a long history of poorly D. Increase renin secretion
controlled diabetes. She presents now with edema, E. Increase aldosteron secretion
hypertension and massive proteinuria, hematuria
and hyperlipidemia. Renal biopsy shows hyaline 2. To cover the above function, what mechanism is
arteriosclerosis of the arterioles. What changes will used?
be present in the glomeruli? A. Increase cardiac output
A. Diffuse proliferative glomerulonephritis B. Increase frequency of the heart
B. Membranous proliferative C. Increase secretion of renin
gomerulonephritis D. Increase secretion of antidiuretic hormone
C. Membranous glomerulosclerosis E. Increase secretion of aldosteron
D. Focal segmental glomerulosclerosis .
E. Nodular glomerulosclerosis 3. Which factor support the above mechanism?
76. A 64-year-old male presents with hematuria. A. Filtration rate of the glomeruli
Examination shows a flank mass and he has B. Active transport of the tubular cell
elevated hemoglobin. Pathology of the surgical C. High osmolality of the interstitium of medulla
specimen shows a clear cell renal cell carcinoma. D. Obligatory water reabsortion
What is the cell of origin of this tumor? E. Osmolality of the filtrate in distal convoluted
A. Glomerulocyte tubule
B. Glomerular endothelium
C. Glomerular epithelium
D. Proximal tubular epithelium (clear ell n 4. If the urinalysis is also done, what will you see about
papilla) the glucose in urine?
E. Histiocyte A. positive ++++
B. positive +++
Distal (klomoford, oncodi) C. positive ++
D. positive+
77. A 32-year-old female presents with headaches. E. negative
Investigation reveals hypertension (180/110
mmHg) and very high level of aldosterone 5. In a kidney laboratory, using a micropipette, we can
measure the hemodynamic pressure of afferent
9
arteriole, pressure in Bowman capsule, and osmotic E. glucose
pressure in glomerular lumen. If the pressure in
afferent arteriole is 40 mmHg, in Bowman capsule 1 10. Your patient, a 45-year-old man, usually lives near
mmHg, the osmotic pressure of glomerulus is 30 the beach, is planning to have a vacation that
mmHg, the effective filtration pressure in glomerulus is: includes a short hike to the top of Mount Everest
A. 71 mmHg (altitude approx. 29.000 feet above sea-level).
B. 69 mmHg You are concern about “mountain sickness”. He
C. 11 mmHg has no other significant medical conditions and
D. 9 mmHg takes no other drugs that would interact with the
E. – 9 mmHg drug you will prescribe for his trip.
Which of the following drug you recommend?
6. A patient taking an oral diuretic for about six A. acetazolamide
months, visits your clinic with elevated fasting and B. amiloride
postprandial blood glucose levels. You check the C. bumetabide
patient’s HbA1C and find it is elevated compared D. furosemid
with normal baseline values obtained 6 months E. spironolactone
ago. You suspect the glycemic problems are
diuretic-induced. Which of the following drug is the 11. A patient with tuberculosis develops bright
most likely cause? orange-red urine after he drank his anti-
A. asetazolamide tuberculosis drugs and calls his physician in a
B. amiloride panic because he is afraid he is bleeding into the
C. chlorothiazide urine. The patient has no other urinary tract
D. spironolactone symptoms. Which of the following medication is
E. triamterene the most likely to produce this side effect?
A. Ethambutol
7. Your patient, a 55 year-old man, with heart failure, B. Isoniazid
unacceptably low cardiac output, and intense C. Pyridoxine
reflex-mediated sympathetic activation of the D. Rifampicin
peripheral vasculature that is attempting to keep E. Streptomycin
vital organ perfusion pressure sufficiently high. The
patient is edematous, and has ascites, because of 12. A 26 year-old young man presents with the
the poor cardiac function and renal compensations symptoms of gonorrhea. This condition is often
for it. Which of the following drug should be associated with infection due to Chlamydia
avoided in this patient? trachomatis.
A. amiloride Which of the following quinolones would be the
B. ethacrynic acid best choice for treating him?
C. hydrochlorothiazide A. ciprofloxacin
D. mannitol B. nalidixic acid
E. spironolactone C. norfloxacin
D. levofloxacin
8. A 63-year-old-man, with a history of heart failure E. ofloxacin
and edema fails to respond adequately to
maximum recommended dosages of 13. A jaundiced 1-year premature infant with elevated
chlorthalidone. free bilirubin is seen in the premature baby
Which of the following is the most likely nursery. The mother had received antibiotic
appropriate for restoring the diuretic response? combination for a urinary tract infection (UTI) 1
A. add with hydrochlorothiazide week before delivery.
B. add with metolazone Which of the following is the most likely cause of
C. replace with furosemide the baby’s kernicterus ?
D. replace with hydrochlorothiazide A. cefixime
E. increasing the dose of B. amoxicillin
chlorthalidone C. azithromycin
D. erythromycin
9. A 75-year-old woman with hypertension is being E. cotrimoxazole
treated with thiazide. Her blood pressure is
decrease to 120/76 mm Hg. After several months 14. A 30 year-old woman with a history of recurrent
on the medication, she complains of being tired urinary tract infection. Since 5 days ago, she felt
and weak. Which of the following substance in dysuria, urgency, and frequency. Culture of urine
analysis of blood indicates low values? sample indicated that the offending organism is
A. calcium Escherichia coli. She receives ciprofloxacin and
B. uric acid the symptoms disappear.
C. potassium Which of the following process is inhibited by
D. sodium ciprofloxacin?
10
A. cell-wall synthesis E. Adrenal gland
B. folic acid synthesis
C. protein synthesis 20. A 70 year old man came to the emergency
D. topoisomerase II department of Karawaci Hospital with
E. DNA polymerase complaint of incomplete urination since 6 months
ago. The urologist performed
15. A 30 year old man with motorcycle accident comes rectal examination and found an enlargement of
to the emergency. In the his prostate.
physical examination, the doctor found that his left Which of the following structure is affected by the
kidney has severe injured. enlargement?
And the doctor suggested operating his kidney. A. urethra pars membranacea
During the removal of a patient’s kidney, B. urethra pars spongiosa
Which of the following structure is the most anterior C. sphichter urethra interna
within the renal sinus? D. urethra pars prostatica
A. Renal arteries E. orificium urethra externa
B. Renal vein
C. Major calyx A 4-year-old-boy was brought to hospital with puffiness
D. Minor calyx around the eyes, especially in the morning and
E. Renal pelvis swelling over the legs. His mother also complaint he
has foamy urine. Urine sample shows protein ++++
16. You wish to examine the hilum of the left kidney (>3.5 per 1.73 m2 per 24 hour), albumin level 1.5g/dL
during surgery. (normal=3.5-5g/dL).
Which of the following structures must be elevated?
A. Stomach 21. What is the principal management in this patient?
B. Suprarenal gland F. Albumin
C. Ascending colon G. Furosemide
D. Duodenum H. Prednisone
E. Liver I. Diet restriction
J. Cyclosporin
17. To elevate the kidney within the renal fascia and
the perirenal fat, which of the 22. During 4 weeks-treatment, the urine sample
following muscles must be reflected or incised from shows protein +.
the fascia? What is your conclusion in this patient?
A. Diaphragm A. Initial responder
B. Psoas B. Late responder
C. Drug resistant
C. Quadratus lumborum D. Drug dependent
D. Transverses abdominis E. Remission
E. Iliacus
A 13 year-old-girl was brought to emergency unit with
18. An 18-year-old man is noted to have probable seizure. Her mother complaint she had fever 2 weeks
testicular cancer. He undergoes ago, but has recovered. Now she has swelling around
surgery. After incising the scrotum, the surgeon the eyes and legs and she has dark urine. From
contemplates the approach physical exam, she is unconscious. Her blood
to the parenchyma of the testes. pressure is 180/120 mmHg. Urine sample shows
Which layer is the most near to the testicular erythrocytes 20/hpf and protein +. Albumin level within
parenchyma? normal limit.
A. Fascia spermatica externa
B. Tunica albuginea 23. What other laboratory findings to confirm the
C. Fascia sprematica interna diagnosis in this patient?
D. Scarpa fascia F. Decreased anti streptolisin O (ASO),
E. Tunica dartos increased DNase-B, decreased C3
G. Increased anti streptolisin O (ASO),
19. A 18-year-old man is noted to have dysuria for decreased DNase-B, decreased C3
several days, and the doctor diagnose that she suffers H. Decreased anti streptolisin O (ASO),
from urinary tract infection. increased DNase-B, increased C3
Which of the following structure of the urinary tract I. Increased anti streptolisin O (ASO),
is the most vulnerable to decreased DNase-B, increased C3
get infection? J. Increased anti streptolisin O (ASO),
A. Kidney increased DNase-B, decreased C3
B. Ureter
C. Urinary bladder 24. What is the most likely mechanism that underlying
D. Urethra this patient’s disease?
11
A. Streptococcal bacteria infection 29. The laboratory test in this patient reveal
B. An inflammation of the kidney tubules haemoglobin 15 g/dL(11.5-15.5 g/dL),
C. Increased permeability of glomerulus hematocrit 46% (35-45%), WBC
membrane 7,000/mm3(5,500-15,500/mm3), platelets count
D. Small pores in the podocytes permit 260,000/mm3 (150,000-400,000 mm3), urea
erythrocyte loss nitrogen 40 mg/dL (5-18 mg/dL),
E. Heparan sulfate mucopolysaccharide is and creatinine 1.7 mg/dL (0.3-0.7 mg/dL).
low so erythrocyte cross the barrier What is the most likely diagnosis in this patient?
F. Acute tubular nephritis
A 5 day old baby was hospitalized due to indirect G. Prerenal acute kidney injury
hyperbilirubinemia. On the third day hospitalization he H. Intrinsic acute kidney injury
had fever and then he was given amoxicillin and I. Postrenal acute kidney injury
gentamicin intravenously. Three days later his urine J. Chonic renal failure
output diminished and the laboratory test revealed
urea nitrogen 25 (3-12) mg/dL, creatinine 0.9 (0.2-0.4) 30. What is the most prior management in this patient?
mg/dL.
25. What is the most likely diagnosis in this patient? A. Bicarbonate
B. Acute Dialysis
F. Urinary tract infection C. Furosemide
G. Sepsis D. Antibiotic
H. Acute renal failure E. Fluid replacement therapy
I. Chronic renal failure
J. Acute tubular nephritis A 10 year old girl was brought to your clinic with
26. What is the most possible cause in this patient? dysuria since 5 days ago. On physical exam she is
healthy with no significant findings. From urine culture
F. Ischemia is found Eschericia coli wih CFU ≥ 100,000.
G. Direct toxic injury 31. What is the most likely diagnosis in this patient?
H. Complex immune response
I. Urinary obstruction F. Acute pyelonephritic
J. Drug hypersensitivity G. Reflux nephropathy
H. Uncomplicated urinary tract infection
27. What is the pathognomonic laboratory finding I. Complicated urinary tract infection
(urinalysis) in this patient? J. Chronic urinary tract infection
A. Increased osmolality
B. Proteinuria 32 .Which of the following factor is unfavorable to
C. Muddy brown casts bacterial growth in patient’s urine?
D. Erythrocyte casts F. A low pH (5.5 or less)
E. Haemoglobinuria
G. A low concentration of urea
A 3 year old boy was brought to emergency unit H. The absence of organic acids from a diet
Karawaci Hospital with severe diarrhea since three days I. A low osmolality
ago. On arrival he is not fully alert. His vital sign are as J. The presence of normal microorganism
follows: blood pressure is 80/60 mmHg, pulse is not
palpable, respiratory rate was 36 breaths per minute, 33. Which of the following antibiotic is the most
and body temperature was 38.4oC. On physical exam appropriate for this patient?
there is sunken anterior fontanel, dry mucous A. Ceftriaxone
membranes, sunken eyes, lack of tears, poor skin B. Cefotaxime
turgor, and the capillary refill more then 2 seconds. The C. Ceftazidime
daily urine volume is less than 400 mL. D. Cotrimoxazole
E. Metronidazole
28. In this patient, the tubular epithelial cells are
particular susceptible to ischemic injury due to:
F. Little charged surface for reabsorption A 55-year-old man refers to ICU after the procedure
G. Low metabolic rate and oxygen open reduction of femur fracture and bleeding intra
consumption operative. At the third day he became apatis, fever
H. Decreased levels of salt reaching the 38,5 Celcius with blood pressure 110/60 mmHg, pulse
distal tubules 55 x/min, respiratory 20x/min, and urine output
I. Loss of polarity that cause redistribution of decreased to 10 cc/hour in 24 hours. At laboratory
membrane proteins examination findings creatinine 4 mg/dL, BUN 250
J. Passive transport systems for ions and mg/dL, sodium 130 mEq/L, chloride 90 mEq/L,
organic acids potassium serum 7.5 mEq/L, blood sugar 200 mg/dL.
From blood gas analysis finding pH 7.1, pCO2 30
mmHg, pO2 90 mmHg,
12
HCO3 18 mEq/L. ECG examination finds widening of kidney region. From sonography findings
QRS wave. multiple hyperechoic shadow from both kidney
with measure about 2 – 5 mm.
34. What is the most critical condition can cause life Which of the following treatment is the most
threatening the patient? effective for this patient?
(A) Nothing for awareness (A) Dietary sodium restriction
(B) creatinine and BUN at very high level
(C) hyperkalemia (B) Dietary calcium supplementation
(D) metabolic acidosis (C) High fluid intake and long-term
(E) Severe hyponatremia urine acidification
(D) High fluid intake and allopurinol
35. What is the best choice of treatment for the clinical (E) Shock wave lithotripsy
situation for the patient?
(A) Food restriction with potassium composition 40. A 40-years-patient arrives at ER department with a
(B) Give K sparing diuretic transthoracic gunshot wound and microscopic
(C) Give regular Insulin 10 U in dextrose 50% hematuria.
intravenous What is the most appropriate supporting diagnostic
(D) Give bicarbonate sodium intravenous technique?
(E) Give forced diuretic (A) Abdominal sonography
(B) Intravenous pyelography
36. After the choice procedure, what is the next (C) Immediate laparatomy
treatment for the patient? (D) Clinical observation on the hospital
(A) Waiting for the general condition of the ward
patient became more better (E) Immediate CT with i.v contrast agents
(B) Give diuretic until the urine output will be
normally 41. You are asked to evaluate a 38-year-old woman
(C) Hemodialysis immediately who has a ten-year history of bipolar disorder. She
(D) Give sodium bicarbonate until pH of has taken lithium carbonate for seven years. For
blood became normal the past two years, she has had severe polyuria
(E) Intubation and ventilator to maintain the (urine volume up to 10 L/24 hr). She refuses to
respiratory discontinue lithium therapy.
What is the most appropriate agent to reduce the
37. According to RIFLE criteria, what is clinical polyuria?
condition of the patient? (A) Amiloride
(A) Risk (B) Chlorpropamide
(B) Injury (C) Furosemide
(C) Failure (D) Hydrochlorothiazide
(D) Loss (E) Acetazolamide
(E) End State Kidney Disease
42. A man 45 years old come with bruise at lower
38. Which of the following statements in true of abdominal and pain. He is suspected with
hematuria caused by bladder cancer? bladder injury,
(A) It is usually accompanied by discomfort and Which of the following statement is true about
painful voiding bladder injuries?
(B) It is intermittent and painless (A) They are present 6% -10% in patients with
(C) It occurs in a minority of patients with bladder pelvic fracture
cancer (B) More than 80% come with urine retention
(D) It commonly causes anemia (C) Delayed diagnosis is the rule
(E) It occurs primarily only in the initial phase of
the urinary stream (D) Bladder contusions commonly result in
gross hematuria
39. A 36-year-old male truck driver is referred to you (E) They are associated with a 50% rate of
for evaluation of recurrent urolithiasis. The urethral tear
patient has passed more than 20 calcium
oxalate stones since age 18. Previous therapy 43. A 51-year-old man, who is hospitalized for
with hydrochlorothiazide and cellulose treatment of Pneumocystis pneumonia, is found to
phosphate has been ineffective. Physical be HIV seropositive. There is no family history of
examination is normal. Laboratory finding: kidney disease or hypertension. Temperature is
Serum creatinine 1.0 mg/dL, Serum uric acid 8.9 38.1 C (100.6 F). Pulse rate is 102 per minute, and
mg/dL, Urine pH 5.0 respirations are 23 per minute. Urinalysis show
Radiographs of the abdomen show smooth renal Protein 3+; 0–2 RBCs; 0–1 WBC/hpf, and Urine
silhouettes measuring 13.6 cm on the right and protein is 6230 mg/24 hr. Ultrasonography
14.2 cm on the left, and no radioopaque at demonstrate large, echogenic kidneys.
13
What pathology feature is demonstrated in the
Kidney biopsy? 48. A 40 years old woman, body weight 50 kg, came to
(A) Postinfectious glomerulonephritis emergency unit with diarrhea (frequency 8
(B) Membranoproliferative times/day) since 2 days ago. Her urine volume was
glomerulonephritis 150 cc/12hours. She was conscious, BP 85/60
(C) Membranous glomerulopathy mmHg, pulse 105 x/min, RR 20 x/min, temperature
(D) Collapsing glomerulopathy 36,5 C. Laboratory results: Hb 12 gr%, Ht 40%,
(E) Minimal change disease WBC 9000, Platelet 263.000, ureum 104,
creatinine 3,7.
44. A 36-year-old woman comes to the emergency What is the most appropriate initial treatment for the
department because of severe, sharp right flank patient?
pain of four hours' duration. During physical A. Give an antibiotic
examination, she cannot lie because of pain. B. Ringer lactate infusion
Temperature is 39.0 C (102.2 F), and pulse rate is C. Give vasopressin
120 per minute. The right flank is tender to deep D. Glucose infusion
palpation. Leukocyte count is 18,000/cu mm. E. Oral rehydration
Serum creatinine is 2.0 mg/dL. Urinalysis shows
protein 1+; 10–20 WBCs, 10–20 RBCs/hpf. 49. A 44 year old man was hospitalized for systemic
Radiograph of the kidneys, ureters, and bladder fungi infection. He was on Amfoterisin B treatmet for
shows a 1.5-cm calcific density in the area of the 7 days. On examination, he conscious, BP 120/80
right mid-ureter. Both kidney silhouettes measure mmHg, pulse 80 x/min, RR 16 x/min, temperature
approximately 12 cm pole-to-pole. 36,7 C. Lab: Hb 13 gr%, HT 37%, WBC 8600,
Ultrasonography shows a hydronephrotic right Platelet 340.000. Urinalysis: RBC 0-1/HPF, WBC 2-
kidney and contracted left kidney. 5/HPF, protein (-), granular cast (+). Ureum 91,
What is the most appropriate management for this creatinin 2, 0. Volume urine 800 cc/24 jam
woman? What is the most likely diagnosis?
(A) Intravenous urography A. Acute Tubular necrosis
(B) Intravenous ketorolac to relax the ureter B. Acute Glomerular Nefritis
(C) Intravenous fluids that allow the stone to
pass
C. Acute Interstitial nephritis
(D) Immediate percutaneous nephrostomy D. Acute Renal Failure
(E) Emergency lithotripsy E. Chronic Renal failure
45. Normal micturition requires that the urinary 50. A young woman went to a physician because
bladder and the urethral sphincter work together pain during urination and frequency. On
as a coordinated unit to store and empty urine. examination, she had has suprapubic pain, but
Which of the following is the effect of no fever. Her urine was cloudy and shows
parasympathetic system in controlling the urinary microscopic evidence of erythrocytes, pyuria,
bladder? and gram-positive bacteria. In the past medical
A. Relaxes external sphincter history, she never had any symptoms like this
B. Relaxes internal spinchter one. Which organism is most likely responsible
C. Relaxes detrussor muscle for this patient’s illness?
D. Contracts detrussor muscle F. Klebsiella pneumonia
E. Contracts anal spinchter G. Pseudomonas aeruginosa
H. Escherichia coli
46. The parasympathetic outflow in the spinal cord I. Proteus mirabilis
occurs at levels: J. Staphylococcus saprophyticus
A. T3,4
B. T 6-10 51. Which urine specimen is most appropriate for
C. S1,2,3 the culture examination for the patient above?
D. S2,3,4 A. Suprapubic
E. L1,2,3 B. Clean catch
C. Straight catheter
47. A 75 year old man had urine incontinent since 3 D. Indwelling catheter
months ago. He had been diagnosed dementia by E. 24 hours urine collection
the doctor since 6 months ago. On examination he
had no prostate problems. What is the cause of his 52. A pregnant woman was complaining fever, pain
urination’s problem? when urinating, blood in her urine, and severe
F. Impaired frontal inhibition lower back pain. The physician submits her urine
G. Impaired pontine inhibition for culture; the next day, the laboratory reports
H. Sympathetic system dysfunction an organisms growth count of greater than
I. Parasympathetic system dysfunction 105/ml of urine. The organism grew well on the
J. Sensoric system dysfunction blood agar and producing β–hemolytic colonies.
14
Which important virulence factor for the 56. What is the most appropriate method to nourish
organism is most likely responsible for this him:
patient’s illness? A. Parenteral nutrition and nil by
A. Capsule mouth
B. P-fimbriae B. Give a fruit juice and add sugar.
C. Endotoxin C. Give milk in enough calories.
D. Phospolipase C D. Temporary no fruits and give
E. Lipooligopolysaccharide blended vegetable.
53. A male patient complained of a sharp pain in his 57. About calcium phosphorus balance in this patient:
suprapubic radiating down to his perineum,
accompanied by hematuria. He reported that he A. He needs milk fortified calcium.
had a few days of urinary urgency, frequency, B. Restrict protein intake
and burning when urinating. On examination, his C. Reduce meat base protein and
temperature was 37,5ºC; other vital sign were nuts.
normal; and he appear to be in pain. A urinalysis D. Reduce protein and give high
shows erythrocytes, leukocytes esterase and a calories.
high pH.
Gram stain demonstrates negative coccobacili.
Which antigenic composition is most likely 58. A 55 year old male complains hematuria at
responsible for this illness? thephysician’s office for 2 weeks.
A. Hemolysin A cystoscopic examinations showed a 2
B. Catalase cm,exophytic,friable mass on the right bladder
C. Fibronectin dome.
D. Opa protein A biopsy of this lesion show findings most
E. Cord factor consistent with a/an
F. Papilloma
A 45 years old man comes to Siloam Hospitals with G. Verrucous carcinoma
low back pain and painful sensation when urinating. H. Transitional carcinoma
Anamnesis: he has a history of kidney stone. Physical I. Squamous carcinoma
examination: pallor, fluctuating pain, blood pressure J. Adenocarcinoma
140/90 mm Hg. Weight 54 kg, Height 167 cm. edema
in low extremity. Lab exams: Hb 13,8 g/dL, WBC 59. A 60 year old male has had multiple recurrent
10.950 mL, Uric acid 7,5 mg/dL, Ureum 86 mg/dL, urinary tract infections with 6 month history of
Creatinin 1,4 mg/dL. dysuria. Rectal toucher by his physician palpates
prostate enlargement.
54. What is the most appropriate nutrition for this What is the most appropiate clinical action to
patient: diagnose carcinoma prostate ?
A. Trans rectal biopsy
A. Restrict water to 1000 ml/day. B. Trans urethral biopsy
B. No restriction for food consistency. C. Transrectal FNAB
C. Drink fruit juice as much as possible. D. Rectal toucher
D. Patients should drink as much as possible. E. Open surgery
E. Ask him to drink while thirst only. Choose dry
foods. 60. A 50 year old male presents with a month long
history of flank pain and hematuria.An USG
55. Protein recommendation: abdominal results a 3 cm bosselated mass,
A. No need to restrict protein protrudes from cortical are in the right
B. Use meat based protein not from the kidney.Cytologic examination reveals malignant
legumes or beans cells. What is most likely biopsy finding for this
C. Limit protein high in uric Acid such as patient?
beans, seeds F. Wilms tumor
D. Limit protein 0,7 gram/kg BB G. Renal Cell Carcinoma
H. Transitional carcinoma
After a year he comes with weakness, nausea, muscle I. Squamous carcinoma
cramp, and itchy. Physical exam: BW 52 kg, edema in J. Undifferentited carcinoma
both legs and palpebras. Blood pressure 170/100
mmHg. Lab exam: Hb 7,6 g/dL, Ureum 196 mg/dL. 61. A 60 year old male experiences the onzet of
Creatinine 3,7 mg/dL. Uric Acid 11,8 mg/dL. Organic headache, naussea ,vomitting fo 1
Phosphorus 11 mg/dL, Calcium 3,4 mg/dL Sodium 147 month.Physical examination reveals
mmol/L, Potassium 5,6 mmol/L. hypertention, subfebril. Pyelogram showed the
affected kidney assymmetrically contracted with
deformity of the calyceal system. Gross of the
15
affected kidney showed contracted and has 65. A 63 year old female patient came with complain
irregular granular surface.The parenchyma is of pain while urinate. She has history of
atrophic and replace by fibrosis. Microscopic recurrent urinary infections. Physical
showed glomerular sclerosis, hyalinization and examination revealed tenderness in CVA region.
atrophi in cortical area. Another areas showed Urinalysis was done and radiology finding
fibrosis, chronic inflammatory cells with reveals large, bilateral "stag horn" calculi.
limpocytes agregation. What kind of stone is likely to be demonstrated
What is the most likely lesions occur in this in this case?
kidney?
A. Calcium oxalate stones
F. Renal cell carcinoma B. Struvite stones
G. Chronic pyelonephritis C. Calcium phosphate stones
H. Acute nephritis D. Uric acid stones
I. Nephrotic syndrome E. Cystine stones
J. Diabetic nephropathy
62. A 5 year old boy child who had a palpable 66. A 55-year-old man is brought to the emergency
abdominal mass brought to hospital by his due to acute onset of severe right-sided flank
mother . The child later had abdominal distention pain. He has a 10-year history of gout. His blood
from bowel obstruction. An USG reveals a 6 cm pressure is 110/80 mm Hg, pulse is 78x/min, and
left renal mass with necrosis and hemorrhage. RR 16x/min. Examination shows normal bowel
Microscopic showed embryonal tissue with sounds and no abdominal tenderness or
abortive of tubular and glomerular structures. masses. Urinalysis shows 40 erythrocytes/hpf.
What is the diagnosis ot this patient? Intravenous pyelography confirms a right
F. Teratoma ureteral calculus.
G. Retinoblastoma Which of the following underlying mechanism is
H. Wilms’tumor responsible for this patient's condition?
I. Hanarthoma A. An increase in urinary pH
J. Hemangioma B. Damage to the epithelial lining of the
ureters
63. A 62-year-old woman presents with hematuria C. Lack of inhibitors of crystal formation
and left flank pain. Physical examination reveals D. Presence of urease-splitting bacteria
costovertebral angle tenderness. Renal E. Urinary supersaturation with uric acid
ultrasound shows marked left hydronephrosis.
Serum creatinine and CBC are normal.
Which of the following diagnostic test is the most A 66 years old, came to Emergency Department with
relevant? shortness of breath as his chief complaint. He also got
A. Renal biopsy nausea, vomiting and loss of appetite since 3 days
B. Renal arteriogram ago. No diarrhea was occurred prior to the symptoms.
C. Palpation for inguinal lymph nodes He was taking un regularly anti hypertension drugs for
D. Antinuclear antibody test 20 years. Physical exam: weight 70 kg, blood pressure
E. IVP 190/110, Pulse 120 times/minute, and Respiratory rate
46 times/minute. Pale conjunctiva, fine crackles +/+
64. A 34-year-old man comes with 3-day history of were heard on the both side of the lung. Heart, liver
low back pain, urinary hesitancy, and pain with and spleen were normal.
urination. He has not had any urethral discharge Lab results: Hb 7.2 g/dl, WBC 7000/mm3, trombosit
or recent sexual contacts. His temperature is 225000/mm3, random glucose 135 mg/dl, Ureum 150
38.2C, and blood pressure is 120/70 mm Hg. mg/dl, Creatinin 5.2 mg/dl. Proteinuria +3
Pulse 96x/min, RR 20x/min. There is no
costovertebral angle tenderness. Abdominal 67. What is the most possible causes of anemia
examination shows no tenderness or masses. based on the case above?
Rectal examination shows no tender prostate A. B6 deficiency
and no masses. B. Erithropoeitin deficiency
Specific gravity 1.030; Blood negative; Glucose C. B12 deficiency
negative; Ketones negative; D. Iron deficiency
Leukocyte esterase positive E. Folic deficiency
What is the most likely diagnosis?
A) Benign prostatic hypertrophy 68. What is the most possible electrolyte level show?
B) Cystitis A. Phosphate decreased
C) Prostatitis B. Calcium increased
D) Pyelonephritis C. Potassium decreased
E) Urolithiasis D. Phosphate increased
16
E. Normal Calcium b. Indentation of superior aspect of bladder
c. Filling defect in the bladder
69. Mrs Jane 35 years old, came to Internal d. Additional shadow of superior aspect of bladder
Department with complain, fatigue and
headache about 1 month . In physical
examination: pale, butterfly rash at her face,
another examination in normal limit. The physician 1. 109 284. The consumption of oxygen by the
suspects that Jane suffers SLE. kidney
What’s the most important lab examination to a. Decreases as blood flow increases
support the diagnosis: b. Is regulated by erythropoietin
a) Complete Blood Count c. Remains constant as blood flow
b) Complement fixation test increases
c) Anti Nuclear Antibody d. Directly reflects the level of sodium
d) Renal function test transport
e) Liver function test e. Is greatest in the medulla
2. 110 285. The anion gap will increase with an
70. According to case no 1, beside protein uria what is increase in the plasma concen-tration of
the most urinalysis result in a. Sodium
this case : b. Potassium
a) Glucosuria c. Chloride
b) Leukocyte uria d. Bicarbonate
c) Hematuria e. Lactate
d) Bilirubinuria 3. 111 288. In the presence of ADH, the filtrate will
e) Crystaluria be isotonic to plasma in the
a. Descending limb of the loop of Henle
71. A 25 years old man has been crashed by a truck. b. Ascending limb of the loop of Henle
The doctor suspects that his kidney is rupture, then he c. Cortical collecting tubule
sent the patient to have IVP. What is the most likely d. Medullary collecting tubule
the appearance of renal parenchymal rupture in IVP? e. Renal pelvis
a. The kidney shows non function 4. 112 289. Sodium reabsorption from the distal
b. Ekstravasation tubule will be increased if there
c. Hydronephrosis is an increase in
d. Normal nephrogram a. Plasma potassium concentration
b. Plasma volume
72. What is the most likely the appearance of renal c. Mean arterial pressure
artery rupture? d. Urine flow rate
a. The kidney shows non function e. Plasma osmolality
b. Ekstravasation 5. 113 290. ADH will be released from the posterior
c. Hydronephrosis pituitary when there is a decrease in
d. Normal nephrogram a.Plasma Na+concentration
b.Plasma volume
73. A 50 years old man comes to the clinic with c.Plasma K+concentration
complain of abdominal colic. The result of IVP d.Plasma pH
demonstrates ureteric dilatation.What is the most e.Plasma Ca2+concentration
likely the problem of ureteric dilatation? 6. 114 291. If 600 mL of water is ingested rapidly,
a. Stone in the pelvicureter junction plasma volume will increase by approximately
b. Kidney cyst a400 mL
c. Stone in the vesicoureter junction b200 mL
d. Renal cell Ca stage I c100 mL
d50 mL
74. A 65 years old man is diagnosed with Prostates e25 mL
hypertrophy. What is the most likely feature appeared 7. 116 293. Renin secretion by the kidney is
in cysptography? increased by
a. Indentation of inferior aspect of bladder a. Increasing mean blood pressure
b. Indentation of superior aspect of bladder b. Increasing glomerular filtration rate
c. Indentation of posterior aspect of bladder c. Increasing sympathetic nerve activity
d. Additional shadow of superior aspect of d. Increasing angiotensin II synthesis
bladder e. Increasing atrial natriuretic hormone
secretion
75 A 45 years old woman is diagnosed with Bladder 8. 117 294. Na+is reabsorbed from the basolateral
stone. What is the most likely feature appeared in surface of the renal epithelial
cysptography? a. cells by
a. Indentation of inferior aspect of bladder b. a Na/H exchange
17
c. Na-glucose cotransport c. Arterial PCO2
d. Na-K pump d. Plasma pH
e. Facilitated diffusion e. Blood volume
f. Solvent drag 16. 125. Decreasing the resistance of the afferent
9. 118 295. Which of the following is most likely to arteriole in the glomerulus of the kidney will
cause an increase in the glomerular filtration decrease
rate? a. The renal plasma flow
a. Contraction of mesangial cells b. The filtration fraction
b. Blockage of the ureter c. The oncotic pressure of the peritubular
c. Release of renin from the capillary blood
juxtaglomerular apparatus d. The glomerular filtration rate
d. Dilation of the afferent arterioles e. None of the above
e. Volume depletion 17. 126. If GFR increases, proximal tubular
10. 119 296. The daily production of hydrogen ion reabsorption of salt and water will increase by a
from CO2 is primarily buffered by process called glomerulotubular balance.
a. Extracellular bicarbonate Contributions to this process include
b. Red blood cell bicarbonate a. An increase in peritubular capillary
c. Red blood cell hemoglobin hydrostatic pressure
d. Plasma proteins b. A decrease in peritubular sodium
e. Plasma phosphate concentration
11. 120. Glomerular filtration rate would be c. An increase in peritubular oncotic
decreased by pressure
a. Constriction of the efferent arteriole d. An increase in proximal tubular flow
b. An increase in afferent arteriolar e. An increase in peritubular capillary flow
pressure 18. 127. Renin release from the juxtaglomerular
c. Compression of the renal capsule apparatus is inhibited by
d. A decrease in the concentration of a. Beta-adrenergic agonists
plasma protein b. Prostaglandins
e. An increase in renal blood flow c. Aldosterone
12. 121. A freely filterable substance that is neither d. Stimulation of the macula densa
reabsorbed nor secreted has a renal artery e. Increased pressure within the afferent
concentration of 12 mg/mL and a renal vein arterioles
concentration of 9 mg/mL. Calculate the
filtration fraction (GFR/RPF). 19. 128. Patients with renal insufficiency develop
a. 0.05 very high plasma concentrations of urea
b. 0.15 (uremia) because of
c. 0.25 a. An increased synthesis of urea by the
d. 0.35 liver
e. 0.45 b. An increased reabsorption of urea by
13. 122. Use the following laboratory data to the proximal tubules
determine the GFR. c. A decreased secretion of urea by the
Urine creatinine concentration =196 mg/mL distal tubules
Plasma creatinine concentration=1.4 mg/mL d. A decreased glomerular filtration rate
Urine flow =1 mL/min e. An increased renal blood flow
The creatinine clearance is approximately 20. 129 . Which one of the following statements
a. 75 mL/min about aldosterone is correct?
b. 98 mL/min a. It produces its effect by activating
c. 125 mL/min cAMP
d. 140 mL/min b. It produces its effect by increasing
e. 196 mL/min distal tubular permeability to sodium
14. 123. The electrically neutral active transport of c. It causes an increased reabsorption of
sodium from the lumen of the kidney occurs in hydrogen ion
the d. TheIt has its main effect on the
a. Proximal tubule proximal tubule
b. Descending limb of the loop of Henle e. It is secreted in response to an
c. Ascending limb of the loop of Henle increase in blood pressure
d. Cortical collecting duct 21. 130 . The effect of antidiuretic hormone (ADH)
e. Medullary collecting duct on the kidney is to
15. In metabolic acidosis caused by diabetic a. Increase the permeability of the distal
ketoacidosis, which of the following would be nephron to water
greater than normal?s b. Increase the glomerular filtration rate
a. Concentration of plasma HCO3− c. Increase the excretion of Na+
b. Anion gap d. Increase the excretion of water
18
e. Increase the diameter of the renal c. Broad casts in urinalysis
artery d. Proteinuria
22. 131 . The glomerular filtration rate will increase e. Hypocalcemia
if 29. 239. Nephrotic syndrome is associated with
a. Sympathetic nerve activity to the a. Excessive renal salt and water loss
kidney increases b. Hyperlipidemia due to lipoprotein
b. The afferent arteriolar resistance excess
increases c. Bleeding due to loss of clotting factors
c. The efferent arteriolar resistance d. Hypothyroidism due to loss of thyroid-
decreases binding globulin
d. The plasma protein concentration e. e. The outer medullary collecting duct
decreases 30. 240. A patient with chronic renal failure will be
e. Urine flow through the urethra is expected to have which of
blocked a. the following findings due to the
23. 132 . Potassium-sparing diuretics inhibit mechanisms described?
Na+reabsorption in the b. Hypercalcemic due to elevated PTH
a. Proximal tubule hormone
b. Thin descending limb of Henle’s loop c. Prolonged bleeding due to decreased
c. Thick descending limb of Henle’s loop synthesis of clotting factors
d. Distal convoluted tubule d. Anemia due to increased red cell
e. Cortical collecting duct destruction
24. 133. Which one of the following values will be e. Hypermagnesemia due to decreased
above normal in a diabetic renal excretion
a. patient with a blood glucose
concentration of 600 meq/L? 31. 241. A high fractional excretion of sodium is
b. Urine flow typically found in
c. Intracellular volume a. Heart failure
d. Plasma sodium concentration b. Urinary tract obstruction
e. Arterial pH c. Acute tubular necrosis
f. Alveolar PCO2 d. Acute glomerulonephritis
25. 134 . Which one of the following will be e. Hepatorenal syndrome
increased in a patient suffering from persistent 32. 242. Which of the following nephron segment is
diarrhea? correctly paired with its function?
a. The filtered load of HCO3− a. Distal tubule and bicarbonate
b. The production of ammonia by the reclamation
proximal tubule b. Loop of Henle and potassium
c. H+secretion by the distal nephron regulation
d. The anion gap c. Proximal tubule and urinary
e. The production of new bicarbonate by concentration
the distal nephron d. Collecting tubule and water regulation
26. 135 . Diuretics, such as acetazolamide, which 33. 243. Which of the following statements is true
produce their effect by in the management of acute renal failure?
a. inhibiting carbonic anhydrase, inhibit a. Metabolic acidosis is fully corrected
the reabsorption of sodium in with bicarbonate
b. The proximal tubule b. Hyperphosphatemia is primarily
c. The thick ascending limb of Henle’s managed with dialysis
loop c. Low-dose dopamine is used to shorten
d. The distal convoluted tubule the duration of renal failure
e. The cortical collecting duct d. Hypervolemia is managed with high-
27. 237. Which is a common finding in acute dose loop diuretics
glomerulonephritis? e. Hyponatremia is corrected by
a. Pulmonary congestion due to volume administration of sodium salts
expansion 34. 244. Which of the following describes bone
b. Hypovolemia due to tubular abnormalities in patients with chronic renal
dysfunction failure?
c. Uniformly progresses to chronic renal a. Osteitis fibrosis cystica is a result of
failure if untreated oversuppression of PTH
d. Urine showing leukocytes and b. Adynamic bone disease is associated
eosinophils with myopathy
28. 238. Which finding is fairly specific for chronic c. Osteomalacia is due to excessive
renal failure? accumulation of magnesium
a. Anemia d. Hyperparathyroidism responds well to
b. Hyaline casts 1,25 dihydroxyvitamin D
19
e. Amyloidosis is similar in etiology to d. Rhabdomyolysis
patients who are not on dialysis 42. 256. Which may cause acute renal failure in
35. 245. Which one of the following statements is patients with nephrotic syndrome?
true concerning hematologic disorders in CRF? a. Dietary protein restriction
a. Resistance to erythropoietin is most b. ACE inhibitors
commonly due to aluminum overload c. Lipid-lowering agents
b. Erythropoietin administration is d. Loop diuretics
associated with worsening 43. 264. A 25-year-old man with flank pain is found
hypertension to have three cysts in each kidney, normal
c. The major cause of death in CRF is hepatic and renal function, and family history is
sepsis not clear. He is most likely to have
d. Abnormal bleeding responds best to a. Autosomal dominant polycystic kidney
platelet transfusion disease
e. Leukocyte function is generally b. Autosomal recessive polycystic kidney
unimpaired disease
36. 246. Which of the following measures has not c. Acquired cystic disease
been shown to retard progression of renal d. Medullary sponge kidney
failure? 44. 266. Which is an accurate statement
a. Aggressive BP control concerning diabetic nephropathy?
b. Decrease in protein intake a. Most patients with type 2 diabetes will
c. ACE inhibitors above other develop this problem
antihypertensives b. It is almost always associated with
d. Erythropoietin for anemia retinopathy in type 1 diabetes
37. 247. In patients with chronic renal failure, which c. ACE inhibition is only indicated for
of the following adaptations are normal? patients with hypertension
a. Fractional excretion of sodium d. Routine dipstick urine should be
increases due to suppression of performed to screen for early disease
aldosterone 45. 267. Which of the following is a secondary
b. Metabolic acidosis due to loss of cause for focal segmental sclerosis?
bicarbonate in the urine a. Hodgkin’s disease
c. Increased potassium loss through b. Colon cancer
extrarenal mechanisms c. HIV disease
d. Decreased fractional excretion of water d. Hepatitis C infection
due to ADH resistance 46. 268. A patient with Crohn’s disease passes a
38. 248. Which of the following serologic finding is kidney stone; the most likely
associated with linear staining of the a. composition is
glomerulus on immunofluorescence? b. Calcium phosphate
a. Anti-GBM antibody c. Uric acid
b. Low complement immune complex d. Struvite
glomerulonephritis e. Calcium oxalate
c. ANCA associated renal disease 47. 277. Hyperkalemia may be caused by
d. Membranoproliferative a. Trimethoprim
glomerulonephritis b. Albuterol
39. 249. Antineutrophil cytoplasmic antibody c. Licorice
(ANCA) is typically present in which systemic d. Cisplatin
disease?
a. Goodpasture’s syndrome 48. 278. A middle-aged patient with an elevated
b. Wegener’s granulomatosis serum creatinine, hypertension, and mild
c. Systemic lupus erythematosus anemia comes to you for evaluation. Urine
d. Thrombotic thrombocytopenic purpura dipstick shows trace protein without red cells or
40. 252. Prerenal azotemia is associated with cellular casts. A 24-h urine collection reveals 5
a. High fractional excretion of sodium g of protein. The most likely etiology is
b. Granular casts in the urine a. Focal segmental sclerosis
c. Use of angiotensin-converting enzyme b. Hypertensive nephrosclerosis
(ACE) inhibitors in unilateral renal c. Amyloidosis
d. artery stenosis d. Multiple myeloma
e. Evolution to acute tubular necrosis if 49. 279. Which of the following is a common cause
untreated of isolated hematuria with isomorphic red cells
41. 254. Leukocytes and white cell casts in the in the urine?
urine are typically seen in a. Alport’s syndrome (hereditary
a. Radiocontrast nephropathy nephritis)
b. Methicillin-induced renal insufficiency b. Thin basement membrane disease
c. Aminoglycoside nephrotoxicity c. Idiopathic hypercalciuria
20
d. IgA nephropathy potential effect of hypovolemia. increase in the
50. 280. A 26-year-old woman with a history of plasma level of which of the following
mitral valve prolapse comes in with 1 week of hormones will bring about the most dramatic
fever that started 3 days after a dental decrease in urinary volume flow rate?
procedure. Her urine contains red cells and her
rheumatoid factor is elevated. Which of the A. angiotensin II
following serologic abnormalities is expected to B. atrial natriuretic peptide
be present? C. PTH
a. Anti-GBM antibody D. aldosterone
b. Low serum complement levels E. ADH
c. Antineutrophil cytoplasmic antibody 3. 56 y.o. woman is diagnosed w/ small cell lung
d. Elevated IgA levels ca. she has paraneoplastic effect from the ca,
51. 440. A 28-year-old previously healthy female, which release of an atidiruetic hormone-like
with no medical history is now 28 weeks agent. which of the following is the most likely
pregnant. She complains of trouble seeing, to be seen?
polyuria, polyphagia, and polydipsia. What is
her diagnosis? A. elevated serum sodium
a. Gestational diabetes mellitus B. elevated serum osmolarity
b. Deep venous thrombosis C. elevated urine sodium
c. Urinary tract infection D. elevated urine cathecolamines
d. Preeclampsia
52. 4-38. A 48-year-old man presents with 4. Histological sections from an abdominal mass
peripheral edema. He has been healthy and that was removed from a 13 month old female
physically active all of his life. His family reveal undifferentiated mesenchymal cell,
history is unremarkable. His blood pressure is immature tubules, and abortive glomerular
normal. On physical examination, the patient is formation. What is the best diagnosis for this
noted to have anasarca. Kidneys are not tumor?
palpable. Urinalysis reveals a moderate A. Dupuytren
amount of proteinuria and “grape B. Ewing
clusters”(oval fat bodies/ maltese appearance) C. Ollier
are seen under light microscopy. Which of the D. Warthin
following is the most likely diagnosis? E. Wilms
a. Glomerulonephritis
b. Rhabdomyolysis 5. A 63 y.o. woman has type II DM. PE positive
c. Nephrotic syndrome for peripheral neuropathy in the feet and
d. Acute interstitial nephritis nonproliferative retinopathy. Urinalysis positive
e. Acute tubular necrosis for proteinuria. which of the following treatment
is most likely to attenuate the course of renal
53. 6-31. A 50-year-old woman com- plains of disease?
leakage of urine when she laughs, coughs, or A. calcium channel blocker
sneezes. After stress incontinence, the most B. ACE inhibitor
common causes of this urinary leakage is C. HMG-CoA inhibitor
a. Detrusor dyssynergia D. dietary carbohydrate restriction
b. Unstable bladder E. weight reduction
c. Unstable urethra
d. Urethral diverticulum 6. Ingesting antacids with and after a meal so
e. Overflow incontinence that gastric pH does not decrease below pH 6
will cause a greater than normal secretion of?
A. gastrin
1. To elevate the kidney within the renal fascia B. secretin
and the perirenal fat, the renal fascia must be C. pancreatic bicarbonate
reflected or incised from the fascia of the? D. cholecystokinin
E. somatostatin
A. diaphragm
B. psoas m. 7. A 16 y.o. girl is reffered to the office because
C. quadratus lumborum m. of chronic diarrhea and weight loss. She is
D. transverse abdominis m. experiencing large volume watery diarrhea
E. iliacus m. that is painless. The symptoms persist even
2. Young man, vomit to point where he become when she’s fasting, and there is no
hypovolemic; as evidence by an relationship to foods or liquids. She’s not on
accompanying decrease in BP and feeling of any medication, and there is no travel history
light-headness. The kidney respond by or other constitutional symptoms. Her PE is
reducing urinary volume flow, thus limiting the
21
normal. Which of the following is the most c. Increase facultative water
likely diagnosis? reabsorbtion
A. partial small bowel obstruction d. Increase secretion
B. partial large bowel obstruction e. Increase aldotesteron secretion
C. osmotic diarrhea 55. To cover above function (no 1), what
D. secretory diarrhea mechanism
E. IBS a. Increase cardiac output
b. Increase frekuensi heart
8. A 42 y.o. overweight but otherwise healthy c. Increase secretion rennin
women present with sudden onset of right d. Increase secretion ADH
upper abdominal colicky pain in 45 minutes e. Increase secretion aldosteron
after meal of ayam goreng. Pain is associated 56. Factor yang mensupport prosess diatas ialah
with nausea and vomiting, and any attempt to a. Filtration late of glomeruli
eat since has caused increased pain. What is b. Active transport of tubular cell
the most likely cause? c. High osmolalitas of interstitium of
A. gastric ulcer medulla
B. cholelithiasis d. Obligatory water reabsorbtion
C. duodenal ulcer e. Osmalalitas of filtrate in distal
D. acute hepatitis contortus tubulus
E. esophageal spasm 57. Pasien dengan heart failure = low CO, perfusi
hanya organ-organ penting. Obat yang tidak
9. A 42 y.o. male, executive, complains of bole diberikan? Thiazide harusnya mannitol
abdominal pain that began about 6 months 58. Pasien dengan heart failure, diberi obat
ago, is constant in nature especially after Cloro..tiazid/mid dengan maximum dose tidak
meals, and located in the upper midabdomen merespon. Diberi apa?furosemid
superior to umbilicus. He also report some 59. Pasien diberikan riazid selama… tahun. Efek
“heartburn” that has been occurring during the yang terjadi decrease kalium
previous year. He as been under a lot of job- 60. Mountain sickness merupakan side effect?
related stress. His stool have changed in color dari?
over the previous 2 months and now are a. czetazolamide->bukan side effect, tp
intermittently dark and tarry in consistency. profilaksis
The physician test the patient stool and find b. Budenamide
occult fecal blood. What is the most likely c. Amyloride
diagnosis? d. Furosemide
A. peptic ulcer e. Spironolactone
B. gastritis 61. Seorang pasien sedang menjalani
C. Chron’s disease pengobatan, pasien complaint Bright orange-
D. gastric polyp red urine merupakan side effect dari obat?
a. Ethambutol
10. A 38 y.o banker with a history of heartburn b. Isoniazid
suddenly experieces excruciating pain in the c. Piridizone
epigastric region of the abdomen. Surgery is d. Rifampisin
performed immediately on admission to ER. e. Streptomisin
There is evidence of an ulcer, which has 62. Pasien mengeluh dengan complaint(symtomp
ruptured through the posterior wall of gonorrhea) treatmentnya?
duodenum. What blood vessel might be a. Apnofloxacin
subject to erosion? b. Nalydixic acid
A. common hepatic c. Norifloxacin
B. left gastric d. LEvofloxacin--
C. splenic e. Ofloxacin
D. superior mesenteric 63. Jaundice 1 yo, increase blirubinn. Ibunya
E. gastroduodenal diberi antibiotic for UTI, baby mengalami
kernicterus ,disebabkan karena obat
cotrimoxazole
KIDNEY 64. 30 yo waman recurrent UTI, culture E coli di
kase aprofloxacin. Penghambat apa?Dna
54. 42 year old man, muscular weakness, mild gyrase
headache after eating salted fish. Small 65. 30 yo man motor accident most anterior within
amount water drink. Usual BP 100/70, Now renal sinus. Renal vein
become 140/90, Na : 150 mEq/ml, K : 3,5, 66. You wish examine the hilum of the left kidney
glucose 150. efek? during surgery. Which pf following structure
a. Increase GFR must be elevated. pancreas
b. Increase obligatory water reabsorbtion a. Stomach
22
b. Suprarenal gland c. Increase permeability of glomerulus
c. Ascending colon membrane
d. Duodenum d. Small pores in podocyte permit rbc
e. Liver pass through
67. To elevate kidney within the renal fascia dan e. Heparansulfat polysaccharide is low,
perirenal fat which of following muscle so rbc could cross the BBB
a. Diafragma 74. Anak di rawat karena indirect bilirubinemia. 3
b. Psoas hari kemudian demam, dan diberi gentamicin
c. Quadratus lumborum dan amoxicilin. Kenaikan BUN dan creatinine.
d. Transverse abdominis Apa mechanisme nya?
e. Illeus a. ATN
68. Laki-laki probable testicular cancer, which b. AIN
layer is the most near to testicular parenchyma c. AKI
a. FS xterna d. CKI
b. Tunica albuginea 75. Kasus no 21 bisa terjadi karena Disebabkan
c. FS interna oleh?
d. Scarpa interna a. Iskemik
e. Tunica dartos b. Toxic injury
69. 18 tahun laki-laki. Dysuria for several day – c. Drug hypersensitivity
UTI. Structure yang vulnerable get infection? d. Immune Complex
a. Kidney 76. 10 tahun, dysuria 5 hari yang lalu. No
b. Ureter significcant finding, urine culture E coli >
c. Bladder 100.000
d. Uretra a. Acute pyelonefritis
e. Adrenal gland b. Reflux nephropathy
70. 70 laki-laki – came to emergency room. c. Uncomplicated UTI
Incomplete urination since 6 bulan lalu. d. Complicated UTI
Structure yang change? e. Chronic UTI
a. Uretral pars membranosa 77. Which of the following factor unfavorable to
b. Uretral pars spongiosa bacterial growth in patients urine
c. Uretral pars prostatica a. A low pH (5,5 or <)
d. Uretral orificium uretra b. Low concentration urea
c. Abscense of organic … a diet
71. During 4 weeks treatment, the urine protein +1 d. Low osmolality
, so the diagnosis : e. Presence of normal microorganism
a. Initial responder 78. Antibiotik untuk UTI first line
b. Lat responder a. Ceftriaxone
c. Drug resistant b. Cefotaxime
d. Drug dependent c. Ceftrazidime
e. Remission d. Cotrimoxazole
72. 13 years old boy comes to ER with seizure, e. Metronidazole
fever 2 weeks but has recovered, swelling in 79. Gfr <15%
the eyes and legs, dark urine. Unconscious , Rifle criteria
bp 180/120. Urine erythrocyte 20/hpf, protein a. Risk
+4, albumin normal. Lab finding b. Injury
a. ASO decrease, dnase-b increase, c3 c. Fail
decrease d. Loss
b. ASO increase, dnase-b decrease, c3 e. ESKD
decrease 80. Pada orang yang mengalami Hematuria in
c. ASO decrease, dnase-b increase, c3 bladder cancer,bagaimana bentuk rasa
increase sakitnya Painless and intermitten
d. ASO increase, dnase-b decrease, c3 81. PH urine = 5. Uric acid = 8,9
decrease a. Lot of fluid + allopurinol
e. ASO increase, anti dnase-b increase, b. Lots of fluid + aciding agent
c3 decrease 82. 40 tahun old male. Thoracic gunwound,
Anti-DNase B is a blood test to look for a substance hematuria. Diagnose?renal?
produced by Group A Streptococcus, the bacteria that 83. Bladder injury. Mostly caused by?trauma
cause strep throat. 84. Polyuria, kasih obat apa?vasopressin
85. Pneumocyctis carinii.. WBC increase etc.,
73. Underlying mechanism dari pertanyaan 19 ? post infection glomerulonefritis
a. Streptococcal infection 86. Pengaruh parasimpatis terhadap bladder
b. Inflammation of kidney tubule contraction of detrusor muscle
87. Parasimpatic S2,3,4 dan cranial 3,7,9,10
23
88. Dementia ,cant holding a pee. frontal lobe area : fibrosis, chronic inflammation cell with
89. Diarrhea 8 hari what the best treatment RL limfosit aggregation
infusion a. RCC
90. 44 tahun laki-laki. Systemic fungi infection on b. Pyelonefrits
amphotericin B treatment for 7 days. c. Nefritis
Conscious, BP 120/80, pulse 80x/min, RR d. NS
16x. 36,7 c, HB 13, HT 37, WBC 8600. e. Diabetic nefropathy
Platelet 340.000, Urinalisis RBC (0-1), WBC 97. 5 years old, pale, abdominal mass. Distention
(2-5), protein -, Granular cast +, Ureum 91, to bowel e=obstraction. USG : 6 cm left renal
creatinin 2,0, urine 800/12 jam mass with necrosis and hemorrhage. Micros :
a. ATN embrional tissue with abbortive tubular and
b. Acute glomerulonefritis glomerular structure Wilms tumor
c. Acute interstitial nefritis 98. 62 tahun, hematuri, left flank pain. PE :
d. ARF costovetebral angle tender. USG : left
e. CRF hydronefrosis. CN and CBC ….
91. Woman, pain during urination and suprapubic a. Renal biopsy
pain, no fever, cloudy urine. Microscopic b. R. ateriogram
hematuria, pyuria, granular +, no past history c. Palpasi lymph node
a. Klebsiella pneumonia d. ANA
b. P. aeruginosa e. IVP
c. E coli 99. Renal trauma, terlindas truk, ginjal kena
d. Proteus mirabilis kerusakan. Parenkimnya rusak. Maka yang
e. S. saphroliticus terjadi pada ginjal tersebut ialah?
92. Which urine specimen most appropriate? a. Ginjal tidak berfungsi
Untuk culture b. Ekstravasasi
a. Suprapubic c. Hidronefrosis
b. Clean catch d. Normal urogram
c. Strecth catheter 100. Injury kena renal artery
d. Indualing catheter 101. Pada pasien dengan pemeriksaan
e. 24 hour urine Radiology menunjukkan ada pelebaran ureter,
93. UTI, beta hemolytic virulence factor maka diperirakan batu berada di?
a. Capsule a. Vesikoureter
b. P fimbrae b. Pelvicc ureter
c. Endotoxin 102. 65, laki-laki, Ada bladder dengan
d. Fosfolipase c perbesaran prostat, kama radiografi. Gimana
e. Lipopolisaccaride
94. UTI, - gram, coccobacillus, high pH.virulence a. penonjolan dari inferior bladder
vactornya? b. Penonjolan dari superior bladder
a. Hemolisin c. Penonjolan dari superior bladder
b. Fibronecrine d. Penonjolan dari posterior bladder
c. Catalase 103. 50 tahun colic. IVP = uteric dilatation.
d. Cordially Kenapa bisa dilatasi?
e. Opaprotein a. Stone in pelvicureter junction
95. Pada orang yang pernah mengalami atau b. Kedney cysts
sedang mengalami kidney stone,tindakan c. Stone in the vesicoureter junction
preventive apa yang diperlukan agar tidak d. Renal cell CA stage 1
terjadi penambahan atau recurrenci dari batu 104. 45 wanita, bladder a stone.
ginjal tersebut? Cystography
a. Restriksi air 100 ml a. Indertaion of inferior aspect of bladder
b. Minum jus yang banyak b. Indertation of superior aspect of
c. Minum sebanyak-banyaknya gabagus bladder
kebanyakan c. Filling defect in the bladder
d. Minum pas haus
e. Makan buah
1. 4yrs boy. Puffy eye in the morning,swelling
96. 60 tahun, headache, nausea, vomit 1 bulan. leg, foamy urine, protein +4, serum albumin
PE : HT, subfebrile. Pyelogram showed 1,5. Principle managementprednison 60mg.
affected kidney asimetri contracted with 2. Urine sample dari +4 ke +1 selama 1 bulan.
deform of calyceal system. Gross of the Patient responsive classification: initial
affected kidney showed contacted dan has responder
irregular granular surface. Parenkim atropi and 3. Boy, severe diarrhea. Not full alert. Bp 80/60,
replace fibrosis. Microscopi : glomerular pulse not palpable, RR 30, temperature
sclerosis, hyalinisasi, atropi incortical. Another 38,4°C, sunken fontanele etc (moderate-
24
severe dehydration). Distal tubular susceptible b. Posterior lobe
to ischemic injury : loss of polarity re c. Lateral lobe
distribution of membrane protein d. Middle lobe
1. During the removal of a patient’s kidney, you
would observe which of the following as being 8. A 48-year-old man is undergoing cystoscopic
most anterior within the renal sinus? examination. As the cystoscope is placed into
a. Renal arteries the urethra through the penile portion, which of
b. Renal vein the following tissue surrounds the urethra?
c. Major calyx a. Prostate pars prostatica
d. Minor calyx b. Corpus spongiosum pars spongiosum
e. Renal pelvis c. Seminal colliculus
d. Spinchter urethrae muclespars
2. You wish to examine the hilum of the right membranacea
kidney during surgery. Which of the following
structures must be elevated and reflected to do 9. A police detective takes a scraping of some
so? stains to be examined for alkaline phosphatase
a. Stomach to assess whether these might be ejaculate.
b. Suprarenal gland What is the source of alkaline phosphatase in
c. Ascending colon the semen?
d. Duodenum a. Prostatic gland
e. Liver b. Bulbourethral glands
c. Seminal vesicles
3. To elevate the kidney within the renal fascia d. Seminal colliculus apparatus
and the perirenal fat, the renal fascia must be
reflected or incised from the fascia of the: 10. A 39 yo woman complains of hematuria and
a. Diaphragm significant flank tenderness. She has a history
b. Psoas muscle of kidney stones. A CT scan depicts the
c. Quadratus lumborum muscle abdominal portion of the ureter lying anterior to
d. Transverse abdominis muscle a muscle. Which of the following is most likely
e. Iliacus muscle to be the name of this muscle?
a. Psoas
4. Which of the following is the male homologue b. Serratus anterior muscle
of the female clitoris? Lebih tepatnya glandula c. Obturator muscle
harusnya d. Rectus muscle
a. Epididimis e. External oblique muscle
b. Vas deferens
c. Penis 11. An individual is known to be suffering from DM.
d. Scrotum Recently, he has developed hypertension. His
doctor suspects that the patient may be
5. The scrotum appears to have a slightly developing renal insufficiency that is leading to
pigmented and wrinkled appearance. What is a reduced glomerular filtration and, as a result,
the explanation for this appearance? hypervolemia and hypertension. The doctor
a. Hyperkeratinized squamous epithelium wishes to evaluate kidney function by
b. The tunica albugenia measuring the GFR. Which is the best
c. The dartos fascia substances for estimating the GFR from the
d. The pampiniform plexus urine?
a. Creatinine
6. An 18-years-old man is noted to have probable b. Para-aminohippuric acid (PAH)
testicular cancer. He undergoes surgery. After c. Urea
incising the scrotum, the surgeon contemplates d. Glucose
the approach to the parenchyma of the testes. e. Sodium
Through which layer must the surgeon incise to
reach the testicular parenchyma? 12. A 21 yo man has been vomiting to the point
a. Buck’s fascia where he has become hypovolemic, as
b. Tunica albuginea evidence by an accompanying decrease in
c. Dartos fascia blood pressure and a feeling of light-
d. Scarpa’s fascia headedness. The kidneys respond by reducing
urinary volume flow, thus limiting the potential
7. A 66-year-old man complains of difficulty extent of hypovolemia. Increase in the plasma
voiding and is noted to have probable BPH. levels of which of the following hormones will
Which of the following prostatic lobes is likely to bring about the most dramatic decrease in
be responsible for these symptoms? urinary volume flow rate?
a. Anterior lobe a. Angiotensin II
25
b. Atrial natriuretic peptide complaining of dizziness, weakness, and
c. PTH nausea. The most likely cause of the patients
d. Aldosterone worsening condition is the development of :
e. ADH a. Metabolic acidosis
b. Hyponatremia
13. A 56 yo woman is diagnosed with small cell c. Hypocalcaemia
carcinoma of the lung. She has a d. Hypokalemia
paraneoplastic effect from the cancer, with e. Hypovolemia
release of an antidiuretic hormone-like agent.
Which of the following is the most likely to be 18. A 35 yo female is noted to have new onset
seen? hypertension that is thought to be due to an
a. Elevated serum sodium aldosterone-secreting adrenal tumor. Which of
b. Elevated serum osmolarity the following is likely to be seen in this patient?
c. Elevated urine sodium a. Hypertension markedly improved with
d. Elevated urine catecholamines furosemid
b. Elevated serum sodium level
14. An individual has adult-onset diabetes. She has c. Elevated serum potassium level
high levels of glucose in the urine and is d. Elevated urinary cortisol level
experiencing a brisk diuresis. The appearance
of glucose in the urine is a consequence of 19. A renal biopsy from an adult who presented
which of the following processes in the proximal with progressive renal failure and hematuria
tubules? reveals linear deposits of IgG within the
a. Inhibition of Na-K ATPase (Na+ pump) glomeruli. What type of autoantibody is most
b. Saturation of the Na-glucose cotransporter likely to be present in this individual?linear itu
c. Saturation of the Na-H exchanger good pasture
d. Stimulation of glucose secretion a. Anti-basement membrane antibodies
e. Stimulation of glycogen breakdown b. Anti-centromere antibodies
c. Anti-double-stranded DNA antibodies
15. A student under stress has been feeling light- d. Antimitochondria antibodies
headed, especially after standing, and has e. Anti-smooth muscle antibodies
developed a brisk diuresis. He has the smell of
acetone on his breath. Upon admission to the 20. Which one of the following combinations of
emergency room he is diagnosed with diabetic signs and symptoms is most consistent with a
ketoacidosis, which is accompanied by extreme diagnosis of nephritic syndrome?
hypovolemia, supposedly because of the brisk a. Hematuria, hypertension, and proteinuria
diuresis. The brisk diuresis is a consequence of b. Massive proteinuria, edema, and
which of the following? hyperlipidemia
a. High levels of glucose in the tubular c. Oliguria, hydronephrosis, and abdominal
fluid/urine rebound tenderness
b. Increased GFR d. Painful hematuria, flank pain, and palpable
c. Suppression of ADH secretion abdominal mass
d. Suppression of aldosterone secretion e. Painless hematuria, polycythemia, and
e. Decreased angiotensin II plasma levels. increased skin pigmentation
16. A hypertensive patient was placed on the 21. A 30 yo female patient presents with a new
diuretic Lasix (furosemid) to increase urinary onset of peripheral edema. PE finds
output. Furosemid, a “high-ceiling” diuretic, is a hypertension & bilateral pedal edema.
potent diuretic because it binds to and inhibits Urinalysis finds massive proteinuria, and
which of the following transport processes? evaluation of her serum finds elevated levels of
a. The Na-glucose co transporter in the cholesterol. A silver stain of a renal biopsy
proximal tubule specimen reveals a characteristic “spike and
b. The Na-K exchange pump in all nephron dome pattern, and electron microscopy finds a
segments uniform deposition of small electron dense
c. The Na-K-Cl cotransporter in the thick deposits in a subepithelial location. Which one
ascending limb of the following immunofluorescence patterns is
d. The Na-Cl cotransporter in the distal most characteristic of this patient’s renal
convoluted tubule disease?membranousgn
e. The Na channel in the cortical collecting a. Granular pattern of IgA and C3
duct. b. Granular pattern of IgG and C3-acute
glomerulonephritis
17. A hypertensive patient is prescribed a loop c. Linear pattern of IgD and C4
diuretic such as Lasix without any supplements. d. Linear pattern of IgE and C4
One week later the patient returns to the clinic e. Linear pattern of IgM and C3
26
a. Amoxycillin
22. A 54 yo man presents with worsening pain on b. ceftriaxone
the left side accompanied by gross hematuria. c. cefixim
Workup finds a 4.5 cm pas in the upper pole of d. Levofloxacin
the left kidney. What is the characteristic e. Gentamycin
histological appearance of the conventional
type of malignancy in this location? 27. A 69 yo woman present with left flank pain and
a. Disorganized groups of immature tubules hematuria. Physical examination suggest a left
b. Sheets of transitional epithelial cells side abdominal mass. CT scan of abdomen
c. Small cells forming numerous papillary reveal 5cm mass in the left kidney. Which of
structures the following laboratory abnormalities might
d. Undifferentiated cells demonstrating also be present?
abortive glomerular formation a. Polycythemia
e. Uniform cells with clear cytoplasm resulting b. Thrombocytopenia
from glycogen c. Hypocalcemia
d. Leukocytosis
23. Histological sections from an abdominal mass e. High rennin hypertension
that was removed from a 13-month-old female
reveal undifferentiated mesenchymal cells, 28. A 60 yo man with heart failure and normal
immature tubules, and abortive glomerular renal function is start of furosemid (Lasix)
formation. What is the best diagnosis for this 80mg/day. She notice a good diuretic
tumor? response every time she take the medication.
a. Dupuytren tumor A few week later, she is feeling unwell
b. Ewing tumor because of muscle weakness and fatigue, but
c. Ollier tumor her heart failure are better. Which of the
d. Warthin tumor following is the most likely explanation for her
e. Wilms tumor muscle weakness?
a. Hyponatremia
24. A 69 yo man has lost a friend to prostate b. Hypernatremia
cancer, and would like to be evaluated for the c. Hypokalemia
disease. He has no urinary symptoms. Which of d. Hyperkalemia
the following test is most likely indicated to e. Anemia
screen him for prostate cancer?
a. Prostate ultrasound 29. A 78 yo man is brought to the hospital
b. Digital Rectal Examination (DRE) because of nausea and vomiting. On the
c. DRE and prostate specific antigen (PSA) physical examination he appear dry, his
d. PSA abdominal is soft, the JVP is not visible. His
e. None of above laboratory test reveal hypernatremia and free
water deficit is approximately 3L. In what part
25. A 63 yo woman has Type II DM, which is well- of the normal kidney is most of the water
controlled. Her physical examination is positive reabsorb from?
for peripheral neuropathy in the feet and a. Colleting duct
nonproliferative retinopathy. A urinalysis is b. Proximal tubulus
positive for proteinuria. Which of the following c. Distal tubulus
treatment is most likely to attenuate the course d. Ascending loop of Henle
of renal disease? e. Descending loop of Henle
a. Calcium channel blockers
b. ACE inhibitors
c. Hepatic hydroxymethylglutaryl coenzyme A 30. A 25 yo woman present with pain on his back
(HMG-CoA) inhibitors and abdomen. She also complain of having
d. Dietary carbohydrate restriction fever, nausea and vomiting. About 1 year ago
e. Weight reduction she has history of renal stone and sometimes
26. A 22 yo woman come to the clinic with sign of she found some solid material in her urine. On
septic shock. The doctor prescribe her to histopathology examination found
consume antibiotic for 2 weeks. After one glomerulosclerosis, thickening of the artery,
week taking the medications she come to the granulation tissue, and limfosit. What is your
doctor with complain of decrease in urine working diagnosis according to the
volume, and edema. On physical examination histopathology?
found that the blood pressure is 140/90, heart a. Nephroblastoma
rate 80, and RR=20. The urinalysis show b. Chronic pyelonephritis
proteinuria and muddy brown cast were found. c. Acute tubular necrosis
What is the most possible antibiotic that can d. Cystitis
cause this symptoms? e. Acute pyelonephritis
27
31. A 63 yo man with an 8 year history of recurrent
severe arthritis in his large toes has dysuria.
He also had pain that comes from his back
that is radiating to the scrotum. Which of the
following mechanism is the most likely
explanation for his symptoms?
a. Uric acid kidney stones
b. ARF
c. Renal parenchymal uric acid crystals
d. Chronic pyelonephritis
e. Acute pyelonephritis
28